摘要訊息 : 建立嚴格的實數體系.

0. 前言

我們在中學的時候已經學過實數了, 但是我們仍然要重新建立實數這個體系. 因為中學中的實數體系並不是嚴謹的, 如果直接以中學的實數為基礎開始學習數學分析, 就會導致很多定理或者定義缺失嚴謹性. 這就好比在地基不穩的地方建樓, 這個樓能安全嗎?

更新紀錄 :

  • 2022 年 6 月 16 日進行第一次更新和修正.

1. 從有理數到無理數

在中學數學中, 我們已經接觸過有理數和無理數. 對於 x^{2} = 2 這樣的方程式, 如果不擴充有理數, 引入無理數, 那麼這樣簡單的方程式都將無解. 再考慮單位長度的正方形, 它的對角線長度並不在有理數的範圍之內. 要證明某個數為無理數, 一般採用歸謬法.

斷言 1. \sqrt {2} 是無理數.

證明 :

任何有理數都可以表示為分數的形式. 不妨假設存在 pq, 使得 \frac {p}{q} = \sqrt {2}, 即 \displaystyle {\left ( \frac {p}{q} \right )^{2} = 2}\frac {p}{q} 是既約的 (無法再約分).

通過變換, 我們有 p^{2} = 2q^{2}, 由於 \frac {p}{q} 是既約的, 因此 pq 都為整數. 於是, p^{2} 為偶數, 從而 p 也是偶數 (若有 a = 2b, 那麼 a 可以被 2 整除, 因此 a 為偶數; 另外, 偶數的平方必定是偶數). 從而 q 是奇數. 設 r 滿足 \displaystyle {p = 2r}. 置換可知 4r^{2} = 2q^{2}, 即 \displaystyle {q^{2} = 2r^{2}}. 於是 q^{2} 也是偶數, q 也為偶數. 這與 \frac {p}{q} 是既約的相矛盾.

綜上所述, \sqrt {2} 是無理數.

\blacksquare

2. 數體

對於一個數的集合 S, 若 S 中的任意兩個數的四則運算, 即加減乘除 (除數不為 0) 的結果仍然在 S 內, 那麼我們稱集合 S 是一個數體 (number field). 換句話說, 數體的運算具有封閉性 (closure property).

常見的數體有三個 : 有理數數體 \mathbb {Q}, 實數數體 \mathbb {R} 和複數數體 \mathbb {C}. 自然數集合 \mathbb {N}, 整數集合 \mathbb {Z} 和無理數集合 \mathbb {R} \setminus \mathbb {Q} 都不是數體.

要說明一個集合不是數體很簡單, 例如對於自然數集合 \mathbb {N}, 有 1 \in \mathbb {N}2 \in \mathbb {N}, 然而兩數之商 \frac {1}{2} \notin \mathbb {N}. 因此, 自然數集合 \mathbb {N}不是一個數體. 另外對於無理數集合, \sqrt {2} - \sqrt {2} = 0 \notin \mathbb {R} \setminus \mathbb {Q}, 因此無理數集合也不是數體.

3. 分割與無理數

我們首先以有理數為基礎, 引入分割的概念.

定義 1. 考慮把全部有理數分成兩個集合 AA' 滿足 :

  1. AA' 都是非空的;
  2. 任意一個有理數在且唯在一個集合中. 即任取 q \in \mathbb {Q}, 必定有 q \in Aq \notin A'q \in A'q \notin A;
  3. 任取 a \in A, a' \in A', 總有 a < a'.

那麼我們稱集合 AA' 定義了一個有理數的分割 (cut), 記為 A|A'. 其中, A 稱為分割的下組 (closed downwards), A' 稱為分割的上組 (closed upwards).

由分割的定義可知, 對於任意 a \in Aa' \in A', 小於 a 的數必定在下組中, 大於 a' 的數必定在上組中.

例題 1.A = \left \{ a : a < 1 \right \}, A' = \left \{ a' : a' \geq 1 \right \}.

闡述 :

由有理數分割的定義可知, A|A' 定義了一個分割.

\blacksquare

斷言 2. 對於分割 A|A' : A = \left \{ a : a < 1 \right \}, A' = \left \{ a' : a' \geq 1 \right \}, 下組 A 中不存在最大的數.

證明 :

任取 a \in Aa > 0. 要證明下組中不存在最大的數, 只需要找到一個正整數 n 使得 \displaystyle {a + \frac {1}{n} < 1} 即可. 對上式進行變換可知 \displaystyle {n > \frac {1}{1 - a}}. 因此, 任意滿足 n > \frac {1}{1 - a}n 都能夠使得 \displaystyle {a < a + \frac {1}{n} < 1} 成立. 因此, 下組 A 中不存在最大的數.

\blacksquare

例題 1'.A = \left \{ a : a \leq 1 \right \}, A' = \left \{ a' : a' > 1 \right \}.

闡述 :

由有理數分割的定義可知, A|A' 定義了一個分割.

\blacksquare

斷言 2'. 對於分割 A|A' : A = \left \{ a : a \leq 1 \right \}, A' = \left \{ a' : a' > 1 \right \}, 上組 A' 中不存在最小的數.

證明 :

任取 a' \in A'. 要證明上組 A' 中沒有最小的數, 僅需要找到一個正整數 n 使得 \displaystyle {a' - \frac {1}{n} > 1} 即可. 對上式變換可得 \displaystyle {n < \frac {1}{1 - a'}}. 因此, 任意滿足 n < \frac {1}{1 - a'}n 都可以使得 1 < a - \frac {1}{n} < a 成立. 故上組 A' 中不存在最小的數.

\blacksquare

例題 2.A = \left \{ a : a^{2} < 2 \right \}, A' = \left \{ a' : {a'}^{2} > 2 \right \}.

闡述 :

由於我們討論的是有理數的分割, 因此由有理數分割的定義可知, A|A' 定義了一個分割.

\blacksquare

斷言 3. 對於分割 A|A' : A = \left \{ a : a^{2} < 2 \right \}, A' = \left \{ a' : {a'}^{2} > 2 \right \}, 下組中不存在最大的數, 上組中也不存在最小的數.

證明 :

任取 a \in Aa > 0. 要證明下組 A 中不存在最大的數, 只需要找到一個正整數 n 使得 \displaystyle {\left ( a + \frac {1}{n} \right )^{2} < 2} 成立即可. 展開上式可得 \displaystyle {a^{2} + \frac {2a}{n} + \frac {1}{n^{2}} < 2 \Rightarrow \frac {2a}{n} + \frac {1}{n^{2}} < 2 - a^{2}}.\frac {2a}{n} + \frac {1}{n^{2}} < \frac {2a}{n} + \frac {1}{n} = \frac {a + 1}{n}. 若 \frac {2a + 1}{n} < 2 - a^{2}, 則有 \displaystyle {\left ( a + \frac {1}{n} \right )^{2} < 2}. 此時, \displaystyle {n > \frac {2a + 1}{2 - a^{2}}}. 因此, 只需要取 \displaystyle {n > \left \lceil \frac {2a + 1}{2 - a^{2}} \right \rceil}, 便有 \left ( a + \frac {1}{n} \right )^{2} < 2. 故下組中不存在最大的數.

\square

同樣地, 任取 a' \in A', 且 a' > 0. 要證明上組 A' 中不存在最小的數, 只需要找到一個正整數 m 使得 \displaystyle {\left ( a' - \frac {1}{m} \right )^{2} > 2} 成立即可. 展開上式可得 \displaystyle {{a'}^{2} - \frac {2a'}{m} + \frac {1}{m^{2}} > 2 \Rightarrow \frac {1}{m^{2}} - \frac {2a'}{m} > 2 - a}.\frac {1}{m^{2}} - \frac {2a'}{m} < \frac {1}{m} - \frac {2a'}{m} = \frac {1 - 2a'}{m}. 若 \frac {1 - 2a'}{m} > 2 - a', 則有 \displaystyle {a' - \frac {1}{m^{2}} > 2} 成立. 變換可得 \displaystyle {m < \frac {1 - 2a'}{2 - a'}}. 因此, 只需要取 m < \left \lfloor \frac {1 - 2a'}{2 - a'} \right \rfloor 就有 \left ( a' - \frac {1}{m} \right )^{2} > 2 成立. 因此上組 A' 中不存在最小的數.

\square

綜上所述, 對於分割 A|A', 下組中不存在最大的數, 上組中也不存在最小的數.

\blacksquare

斷言 4. 不存在這樣一個分割 A|A', 它既滿足上組中存在最大的數, 由滿足下組中存在最小的數.

證明 :

設分割 A|A' 滿足 A 中有最大的數 a, A' 中有最小的數 a'. 那麼根據分割的定義, 自然有 \displaystyle {a < a'}. 如果 aa' 存在無窮小數, 且小數部分前 k 位相同, 那麼如果整數部分相同, 這兩個數的大小由小數部分的第 k + 1 位決定. 現在取 a 的前 k + 1 位, 在第 k + 2 位任意添加一個十以內的非零整數, 記這個數字為 c. 那麼顯然有 \displaystyle {a < c < a'} 成立. 這樣, c \notin Ac \notin A'. 於是, A|A' 不滿足分割的定義, 和假設存在矛盾.

綜上所述, 不存在這樣一個分割 A|A', 它既滿足上組中存在最大的數, 由滿足下組中存在最小的數.

\blacksquare

那麼現在由斷言 2, 斷言 2', 斷言 3斷言 4 可以得到, 有理數的分割存在三種類型 :

  • 在下組 A 中沒有最大的數, 而在上組 A' 中有最小的數;
  • 在下組 A 中有最大的數, 而在上組 A' 中沒有最小的數;
  • 在下組 A 中沒有最大的數, 同時在上組 A' 中沒有最小的數.

對於有理數的分割來說, 前面兩種類型的分割由有理數 r 產生. r 或者是下組中最大的數, 又或者是上組中最小的數. 我們稱有理數 r 為有理數分割 A|A'界數 (bound number), 或者有理數分割 A|A' 定義了有理數 r. 對於第三種情形, 我們規定這種類型的分割定義了某個無理數 (irrational number) \alpha, 這個 \alpha 替代了本在有理數中無法找到的界數. 根據中學所學的知識, 不難得到例題 2 中的分割 A|A' 定義了 \sqrt {2}, 或者說是 \sqrt {2} 是分割 A|A' 的界數.

Tip : 這樣去引入無理數是嚴謹的.

對於界數分配到下組中還是上組中, 這並沒有明確規定. 但是為了明確起見, 沒有特殊說明的情況下, 我們規定 : 若是提及了由數 r 產生的分割, 我們總是將 r 歸入到上組中.

有理數和無理數一起組成了一個新的集合, 這個集合便是實數 (real number). 實數是數學分析乃至整個數學的基本概念之一.

4. 實數集合的有序化

定義 2. 設分割 A|A'B|B' 分別定義無理數 \alpha\beta. 若 A = BA' = B', 則稱 \displaystyle {\alpha = \beta}.

定義 3. 設分割 A|A'B|B' 分別定義無理數 \alpha\beta. 若 B \subset A, 則稱 \displaystyle {\alpha > \beta}.

顯然地, 定義 2定義 3 都適用於有理數. 對於定義 2, 我們可以將定義簡化, 去處 A' = B' 這一個條件. 因為根據分割的定義, 下組相同, 上組自然就相同.

推論 1. 若分割 A|A', B|B'C|C' 分別定義 \alpha, \beta\gamma. 若有 \alpha > \beta, \beta > \gamma, 那麼有 \displaystyle {\alpha > \gamma}.

證明 :

\alpha > \beta 可知, B \subset A; 由 \beta > \gamma 可知, C \subset B. 於是有 \displaystyle {C \subset A}. 根據定義 2, 自然有 \alpha > \gamma.

\blacksquare

推論 1 類似可得, 若有 \alpha < \beta\beta < \gamma, 那麼有 \alpha < \gamma. 因此, 任意兩個實數 \alpha\beta 之間有三種關係 :\displaystyle {\alpha = \beta, \alpha > \beta \text { 和 } \alpha < \beta}.

引理 1. 不論 \alpha\beta 是兩個怎樣的實數, 若 \alpha > \beta, 那麼總能找到一個這樣的實數 r, 使得 r 介於 \alpha\beta 之間, 即 \displaystyle {\alpha > r > \beta}. 甚至 r 可以是有理數, 並且存在無數個這樣的 r.

說明 :

由於 \alpha > \beta, 因此定義 \alpha 的分割的下組 A 包含了定義 \beta 的分割的下組 B, 即 B \subset A. 那麼自然可以在 A 中找到一個實數, 甚至是有理數 r, 其滿足 r < \alpha, r \notin Br \in B. 其中, B' 是定義 \beta 的分割的上組. 那麼對於 r, 有 \displaystyle {\alpha > r \geq \beta}.斷言 3 可知, A 中沒有最大的數. 因此為了取消等號, 只需要適當放大 r 即可.

\blacksquare

引理 2.\alpha\beta 是兩個給定的實數, s's 是兩個滿足 s' > s 的有理數. 任取 \varepsilon > 0, 若 \alpha\beta 總能夾在 s's 中間, 即 \displaystyle {s' \geq \alpha \geq s \text { 且 } s' \geq \beta \geq s}. 其中, s' - s < \varepsilon. 那麼必定有 \displaystyle {\alpha = \beta}.

證明 :

我們使用歸謬法進行證明. 假設 \alpha > \beta. 由引理 1 可知, 可以在 \alpha\beta 之間插入兩個有理數 rr', 使得 \displaystyle {\alpha > r' > r > \beta} 成立. 由於 s' \geq \alpha\beta \geq s, 結合上式可以得到 \displaystyle {s' > r' > r > s}. 顯然又有 \displaystyle {s' - s > r' - r}. 若取 \varepsilon = r' - r, 則與引理條件矛盾.

對於 \alpha < \beta 的情況類似可證.

綜上所述, \alpha = \beta.

\blacksquare

5. 實數的無盡小數表示法

設實數 \alpha 由分割 A|A' 定義, 它既不是整數, 也不是任何有限十進制小數. 現在考慮 \alpha 的近似. 首先在 A 中找到一整組 M, 在 A' 中找到一整數 N. 逐一增大 M, 逐一減小 N, 直到找到一組相鄰整數 CC + 1, 使得 \displaystyle {C < \alpha < C + 1}. 其中, M = C, N = C + 1. 這裡, C 可以是整數, 負數或者零. 其次, 我們將區間 [C, C + 1] 分成十等份 : \displaystyle {C.1, C.2, ..., C.9}. 那麼 \alpha 落且唯落在其中一個部分. 於是, 我們得到兩個相差為 \frac {1}{10} 的有理數, 即 C.c_{1}C.c_{1} = \frac {1}{10}, 使得 \displaystyle {C.c_{1} < \alpha < C.c_{1} + \frac {1}{10}}. 繼續使用類似的步驟, 在確定了 n - 1 個數字 C.c_{1}c_{2}...c_{n - 1} 後, 我們使用不等式 \displaystyle {C.c_{1}c_{2}...c_{n - 1}c_{n} < \alpha < C.c_{1}c_{2}...c_{n - 1}c_{n} + \frac {1}{10^{n}} \ \ \ \ \ \ \ \ \ \ (\mathrm {I})} 來確定第 n 個數字 c_{n}.

於是在求數 \alpha 的十進制小數近似值的過程中, 我們作出了整數 C 與一無窮序列 c_{1}, c_{2}, ..., c_{n}, ..., 由這些數字組成的無盡小數記為 \displaystyle {C.c_{1}c_{2}...c_{n}...}. 它可以看作實數 \alpha 的一種表示.

有盡十進制小數, 包括整數, 它是一種特殊情形. 不過, 我們仍然可以採用類似的方法來確定 C, c_{1}, c_{2}, ..., c_{n}, .... 只不過, 這比無盡小數更一般 :\displaystyle {C.c_{1}c_{2}...c_{n} \leq \alpha \leq C.c_{1}c_{2}...c_{n} + \frac {1}{10^{n}}}. \ \ \ \ \ \ \ \ \ \ (\mathrm {II}) 因此, 當 n 大到一定程度, \alpha 會和包含它的區間某一端重合. 此時, 上式中的等號便會成立, 即某一端出現等式.

對於有限小數 \alpha = C.c_{1}c_{2}...c_{n}\ (c_{n} \neq 0), 記 c_{n}' = c_{n} - 1, 則 \alpha 可以表示為 \displaystyle {\alpha = C.c_{1}c_{2}...c_{n}000... = C.c_{1}c_{2}...c_{n}'999...} 對於 \alpha < 0 的情形, 還有另一種表示方法. 記 B = |C| + 1, b_{1} = 0 - c_{1}, b_{2} = 9 - c_{2}, ..., b_{n} = 10 - c_{n}, b_{n}' = b_{n} - 1, 則 \alpha 可以表示為 \displaystyle {\alpha = B.b_{1}b_{2}...b_{n} = B.b_{1}b_{2}...b_{n}000... = B.b_{1}b_{2}...b_{n}'999...}

對於無限小數 \alpha, 我們稱 C.c_{1}c_{2}...c_{n}\alpha偏小近似值 (lower approximate value), C.c_{1}c_{2}...c_{n} + \frac {1}{10^{n}}\alpha偏大近似值 (upper approximate value). 這兩個近似值的差為 \frac {1}{10^{n}}. 這個差會隨著 n 的增大小於任意的有理數 \varepsilon > 0. 事實上, 不超過 \frac {1}{\varepsilon} 的正整數僅存在有限多個. 因此, 不等式 \frac {1}{10^{n}} \leq \frac {1}{\varepsilon}, 或者等價地, \frac {1}{10^{n}} \geq \varepsilon 只能被有限多個 n 所滿足; 對於其餘一切 n 值, 都有 \displaystyle {\frac {1}{10^{n}} < \varepsilon}. 根據引理 2 可知, 凡是不同於 \alpha 的數 \beta 都無法滿足 \displaystyle {C.c_{1}c_{2}...c_{n} \leq \beta \leq C.c_{1}c_{2}...c_{n} + \frac {1}{10^{n}}}. 從而, \beta 的無盡小數的表示形式也不同於 \alpha.

由此可見, 在特別情形下, 不等於任何有限小數的數, 其表示形式不能由 0 或者 9 作循環節. 因為每一個以 09 作循環節的小數都表示一個有限小數.

任取一個無盡小數 C.c_{1}c_{2}...c_{n}..., 則存在一個實數 \alpha 使得 \alpha = C.c_{1}c_{2}...c_{n}... 顯然, 只要讓 \alpha 滿足不等式 (\mathrm {II}) 即可. 為此, 我們引入簡短的記號 : \displaystyle {C_{n} = C.c_{1}c_{2}...c_{n}, C_{n}' = C.c_{1}c_{2}...c_{n} + \frac {1}{10^{n}}}. 我們注意到對於整數 m, 當 m = n 時, 由 C_{n} < C_{m}'; 當 m > n 時, 必有 C_{n} < C_{m}'; 當 m < n 時, 也有 C_{n} < C_{m}', 因為 C_{m} + \frac {1}{10^{m}} > C_{m}. 因此, 對於任意整數 m, 都有 C_{n} < C_{m}'. 現在, 在有理數數體中作一個分割 : 所有大於 C_{n} 的有理數 (例如所有 C_{m}') 歸入上組 A' 中, 其餘一切有理數 (包括 C_{n} 自身) 歸入下組 A 中. 不難驗證, 分割 A|A' 定義了一個實數 \alpha. 事實上, 因為 \alpha 是夾在兩組中間的界數, 所以有 \displaystyle {C_{n} \leq \alpha \leq C_{m}'}.

綜合上面的討論, 任何實數都可以看作無盡小數, 而循環的小數都是有理數, 不循環的小數便是無理數.

以後, 我們需要利用有理數 aa' 來逼近實數 \alpha : \displaystyle {a < \alpha < a'}. 它們的差可以小於任意小的有理數 \varepsilon > 0, 即 a' - a < \varepsilon. 對於有理數而言, aa' 顯然存在; 對於無理數而言, 可以採用足夠大的 n 取到的 C_{n}C_{n}' 作為 aa'.

6. 實數集合的連續性

對有理數集合進行分割是我們引入實數的基礎, 通過分割我們發現某些分割的上下組中不存在最值, 即有理數之間存在空隙. 這個特性我們稱為有理數的不完備性 (incompleteness). 正是有理數的不完備性, 為無理數的引入提供了根據, 那麼自然地, 我們希望探索實數集合是否存在空隙, 即實數集合是否為完備的.

與有理數相似, 我們考慮實數集合的分割.

定義 1. 考慮把全部實數分成兩個集合 AA' 滿足 :

  1. AA' 都是非空的;
  2. 任意一個實數在且唯在一個集合中. 即任取 q \in \mathbb {Q}, 必定有 q \in Aq \notin A'q \in A'q \notin A;
  3. 任取 a \in A, a' \in A', 總有 a < a'.

那麼我們稱集合 AA' 定義了一個實數分割, 記為 A|A'. 其中, A 稱為分割的下組, A' 稱為分割的上組.

定理 1. (Dedekind 基本定理) 對於實數集合的任何分割 A|A', 存在產生這個分割的實數 \beta, 使得 \beta 或者是下組 A 中最大的數, 又或者是上組 A' 中最小的數.

證明 :

B \subset AA 中的有理數集合, B' \subset A'A' 中有理數的集合. 於是, B|B' 構成了一個分割, 並且定義了某個實數 \beta. 而 \beta 或是落在了 A 中, 或是落在了 A' 中. 假設 \beta \in A, 於是我們便要證明 \betaA 中最大的數. 我們使用歸謬法進行證明. 假設在 A 中存在一個 \alpha_{0}, 使得 \alpha_{0} > \beta. 根據引理 1, 必定存在某個數 r, 使得 \displaystyle {\alpha_{0} > r > \beta}r 為有理數 (根據引理 1 顯然可以找到), 由於 \alpha_{0} \in A, 則 r \in A, 同時 r \in B. 然而由於 B|B' 定義 \beta, 因此對於任意 \beta_{0} \in B, 必然有 \beta_{0} \leq \beta. 若 \alpha_{0} 確實存在, 則有理數 r 也必定存在, 此時 B|B' 就無法定義 \beta, 和假設產生矛盾. 因此, \beta 是下組 A 中最大的數.

\square

類似地, 假設 \beta 不是上組 A' 中最大的數, 那麼存在 r_{0} \in A', 使得 r < \beta. 此時, 也必定可以找到一個有理數 r' \in B' 使得 \displaystyle {r < r' < \beta}. 那麼分割 B|B' 同樣無法定義 \beta. 故假設不成立.

\square

綜上所述, 對於實數集合的任何分割 A|A', 存在產生這個分割的實數 \beta, 使得 \beta 或者是下組 A 中最大的數, 又或者是上組 A' 中最小的數.

\blacksquare

定理 1描述了實數集合的完備性, 或者稱為連續性 (continuity). 這個特性是有理數集合所不具備的.

斷言 2, 斷言 2', 斷言 3斷言 4 針對實數集合的分割同樣成立.

7. 數集的界

對於任意有限或者無限集合, 用 x 表示集合中的任一數, 於是 x 可以作為集合元素的標準記號; 數 x 的集合本身可以用 \mathscr {X} = \left \{ x \right \} 來表示.

定義 4. 如果對於所考慮的集合 \left \{ x \right \}, 若存在這樣的數 M, 使得一切的 x \leq M, 我們便說這個集合是上有界的 (upper bounded), 或者說這個集合是由 M 限制且上有界的. 而 M 本身是集合 \left \{ x \right \} 的一個上界 (upper bound).

定義 4'. 對於所考慮的集合 \left \{ x \right \}, 若存在這樣的數 m, 使得一切的 x \geq m, 則稱這個集合是下有界的 (lower bounded), 或者說這個集合是由 m 限制且下有界的. 而 m 本身是集合 \left \{ x \right \} 的一個下界 (lower bound).

一個上有界的集合可以同時是下有界的, 也可以不是下有界的; 反之亦然. 例如, A = \left \{ a : x > 0 \right \} 便是下有界的, 任何小於或者等於 0 的數都可以作為 A 的下界, 而 A 並非上有界的.

若某個集合不是上有界的, 我們便取以一個廣義的數 +\infty 作為其上界; 若某個集合不是下有界的, 我們便取另一個廣義的數 -\infty 作為其下界. 對於任意有限實數 \alpha, 我們總有 \displaystyle {-\infty < \alpha < +\infty}.

定義 5. 對於一個上有界的集合 \mathscr {X} = \left \{ x \right \}, 它必定存在無限多個上界, 所有上界中最小的那個數稱為這個集合的最小上界 (supremum), 記為 \displaystyle {M^{*} = \sup {\mathscr {X}} = \sup \left \{ x \right \}}.

定義 5'. 對於一個下有界的集合 \mathscr {X} = \left \{ x \right \}, 它必定存在無限多個下界, 所有下界中最大的那個數稱為這個集合的最大下界 (infimum), 記為 \displaystyle {m^{*} = \inf {\mathscr {X}} = \inf \left \{ x \right \}}.

定理 2. 如果集合 \mathscr {X} = \left \{ x \right \} 是上有界或者下有界的, 那麼它必定存在最小上界或者最大下界.

論述 :

我們以上有界的情況為例進行論述, 嚴格證明需要引入實數的定義才可以實現. 我們目前所說的實數知識有理數與無理數兩個集合的並.

首先假定在集合 \mathscr {X} 中可以找到最大的數 \bar {x}. 因此, 對於任意 x \in \mathscr {X}, 必定有 x \leq \bar {x}. 也就是說, \bar {x}\mathscr {X} 的一個上界. 另一方面, \bar {x} \in \mathscr {X}; 因此, 對於任何上界 M, 有 \bar {x} \leq M. 因此, 此時 \bar {x} 是集合 \mathscr {X} 的最小上界.

再假設集合 \mathscr {X} 中沒有最大的數. 由於集合 \mathscr {X} 是上有界的, 於是我們將所有上界 \alpha' 歸入 A 中, 其餘實數 \alpha 歸入 A 中. 在這個分法下, 任何 x \in \mathscr {X} 都落在 A 中, 即 x \in A. 由於集合 \mathscr {X} 中沒有最大的數, 於是可以斷定 AA' 都不為空. 根據分割的定義, 這個分法構成了一個分割 A|A', 它定義了一個實數 \beta. 根據我們的分法, 任意 \alpha \in A 都無法使得不等式 \displaystyle {\alpha > \beta} 成立. 而 A 中沒有最大的數, 根據定理 1, \betaA' 中最小的數. 所以, \beta 便是集合 \mathscr {X} 的最小上界.

\blacksquare

我們規定, 若集合 \mathscr {X} = \left \{ x \right \} 不是上有界的, 則稱其最小上界為 +\infty, 即 \displaystyle {\sup {\mathscr {X}} = \sup \left \{ x \right \} = +\infty}; 若集合 \mathscr {X} = \left \{ x \right \} 不是下有界的, 則稱其最大下界為 +\infty, 即 \displaystyle {\inf {\mathscr {X}} = \inf \left \{ x \right \} = -\infty}.

推論 1.M^{*} 是集合 \mathscr {X} = \left \{ x \right \} 的最小上界, 那麼對於一切 x, 有 \displaystyle {x \leq M^{*}}. 現任取 \alpha < M^{*}, 則在集合 \mathscr {X} 中, 必定可以找到一個 x' 使得 \displaystyle {x' > \alpha}.

推論 1'.m^{*} 是集合 \mathscr {X} = \left \{ x \right \} 的最大下界, 那麼對於一切 x, 有 \displaystyle {x \leq m^{*}}. 現任取 \alpha < m^{*}, 則在集合 \mathscr {X} 中, 必定可以找到一個 x' 使得 \displaystyle {x' < \alpha}.

推論 1推論 1' 根據引理 1 是顯然可以得到的.

推論 2. 對於集合 \mathscr {X} = \left \{ x \right \}, 若對於一切 x 滿足 x \leq M, 則 \displaystyle {\sup {\mathscr {X}} = \sup \left \{ x \right \} \leq M}.

推論 2'. 對於集合 \mathscr {X} = \left \{ x \right \}, 若對於一切 x 滿足 x \geq m, 則 \displaystyle {\inf {\mathscr {X}} = \inf \left \{ x \right \} \geq m}.